Series to represent alternate between 1 and -1

  • Thread starter Thread starter foo9008
  • Start date Start date
  • Tags Tags
    Series
Click For Summary
The discussion centers on the representation of a series where coefficients for even indices are zero, while odd indices alternate between positive and negative values. The author questions the correctness of using the term (-1)^(n+1) for odd n, arguing that it leads to inconsistencies, particularly when n=3 results in a positive value instead of the expected negative. A proposed clarification involves redefining n to account for odd indices as n = 2n' - 1, where n' is a positive integer. This adjustment allows for a more accurate representation of the series, ensuring that the sign alternates correctly for odd terms. The conversation emphasizes the importance of precise definitions in mathematical series representations.
foo9008
Messages
676
Reaction score
4

Homework Statement


we know that a2 , a4 ,a6 (even number ) = 0 , but when a1 , a3 , a5 (odd numbers) , the answer of an alternate between positive and negative ... in the second circle , the author represent it with (-1)^(n+1) , i don't think this is correct , this is because when n=3 , an = -2/ 3pi when n=3 , [ (-1) ^(3+1 ) ] = positive 1 , not negative 1 ! can someone explain on this ?
OW5ijzU.jpg

o9H3EwT.jpg


Homework Equations

The Attempt at a Solution

 
Physics news on Phys.org
I think that the author redefined n. To make it clear, you start off assuming that

f(x) = a_0 + \sum_n a_n cos(nx)

Then, for this particular problem, you find that for n > 0, then a_n = 0 unless n is odd. If n is odd, then that means that n can be written as:

n = 2n'-1

where n' = 1, 2, 3, ...

So the term -\frac{2}{\pi} \frac{cos(3x)}{1} corresponds to n=3, but it corresponds to n' = 2. In terms of n', the general term is

\frac{2}{\pi} \frac{cos((2n'-1)x)}{2n'-1} (-1)^{n'+1}
 
Question: A clock's minute hand has length 4 and its hour hand has length 3. What is the distance between the tips at the moment when it is increasing most rapidly?(Putnam Exam Question) Answer: Making assumption that both the hands moves at constant angular velocities, the answer is ## \sqrt{7} .## But don't you think this assumption is somewhat doubtful and wrong?

Similar threads

Replies
3
Views
3K
  • · Replies 1 ·
Replies
1
Views
2K
  • · Replies 10 ·
Replies
10
Views
9K
  • · Replies 5 ·
Replies
5
Views
2K
  • · Replies 8 ·
Replies
8
Views
2K
  • · Replies 2 ·
Replies
2
Views
1K
Replies
1
Views
2K
  • · Replies 2 ·
Replies
2
Views
2K
Replies
7
Views
2K
  • · Replies 8 ·
Replies
8
Views
2K